ML13297A334

From kanterella
Revision as of 14:28, 11 April 2019 by StriderTol (talk | contribs) (Created page by program invented by StriderTol)
Jump to navigation Jump to search
2013-301 Final Administrative Documents
ML13297A334
Person / Time
Site: Sequoyah  Tennessee Valley Authority icon.png
Issue date: 10/18/2013
From:
NRC/RGN-II
To:
Tennessee Valley Authority
References
Download: ML13297A334 (91)


Text

1 ES-401 Written Examination Review Worksheet Form ES-401-9 Q# 1. LOK (F/H) 2. LOD (1-5) 3. Psychometric Flaws

4. Job Content Flaws
5. Other 6. B/M/N 7. U/E/S 8. Explanation Stem Focus Cues T/F Cred. Dist. Partial Job-Link Minutia#/ units Back-ward Q= K/A SROOnly Instructions [Refer to Section D of ES-401 and Appendix B for additional information regarding each of the following concepts.]
1. Enter the level of knowledge (LOK) of each question as either (F)undamental or (H)igher cognitive level. 2. Enter the level of difficulty (LOD) of each question using a 1 - 5 (easy - difficult) rating scale (questions in the 2 - 4 range are acceptable). 3. Check the appropriate box if a psychometric flaw is identified:
  • The stem lacks sufficient focus to elicit the correct answer (e.g., unclear intent, more information is needed, or too much needless information).
  • The stem or distractors contain cues (i.e., clues, specific determiners, phrasing, length, etc).
  • The answer choices are a collection of unrelated true/false statements.
  • The distractors are not credible; single implausible distractors should be repaired, more than one is unacceptable.
  • One or more distractors is (are) partially correct (e.g., if the applicant can make unstated assumptions that are not contradicted by stem). 4. Check the appropriate box if a job content error is identified:
  • The question is not linked to the job requirements (i.e., the question has a valid K/A but, as written, is not operational in content).
  • The question requires the recall of knowledge that is too specific for the closed reference test mode (i.e., it is not required to be known from memory).
  • The question contains data with an unrealistic level of accuracy or inconsistent units (e.g., panel meter in percent with que stion in gallons).
  • The question requires reverse logic or application compared to the job requirements. 5. Check questions that are sampled for conformance with the approved K/A and those that are designated SRO-only (K/A and license level mismatches are unacceptable). 6. Enter question source: (B)ank, (M)odified, or (N)ew. Check that (M)odified questions meet criteria of ES-401 Section D.2.f. 7. Based on the reviewer's judgment, is the question as written (U)nsatisfactory (requiring repair or replacement), in need of (E)ditorial enhancement, or (S)atisfactory? 8. At a minimum, explain any "U" ratings (e.g., how the Appendix B psychometric attributes are not being met).

2 Q# 1. LOK (F/H) 2. LOD (1-5) 3. Psychometric Flaws

4. Job Content Flaws
5. Other 6. B/M/N 7. U/E/S 8. Explanation Stem Focus Cues T/F Cred. Dist. Partial Job-Link Minutia#/ units Back-ward Q= K/A SROOnly1 H 2 X X Y M E K/A 007EK1.02 1. Q=K/A, Q=RO level 2. Remove the statement "to satisfy shutdown margin". Not needed for the questions, potential teaching in the statement.
3. Recommend question to be Based on the given conditions, iaw ES 0.1 Emergency boration __________.
4. Last bullet should state Tave is 539 degrees and stable. Remove any question whether temperature "stabilizing" means higher than the emergency boration requirement.

04/18/2013

Facility Licensee modified Q as requested by NRC. Q appears SAT at this time. 2 H 2 X Y B E K/A 008AK2.01 1. Q=K/A, Q=RO level 2. Distracter C and D are in essentially the same location and provide the same thermodynamic properties. With two distracters being the same, a candidate would have a 50/50 pick between A and B. Need to replace C or D with another plausible location.

3. Used on the Watts Bar 08/2010 exam.

04/18/2013

Facility Licensee modified one distracter to provide an alternate location. Q appears SAT at this time. 3 H 2 X X Y B U K/A 009EK1.02 1. Q=K/A, Q=RO level 2. The bullet in the initial conditions that states, "the next step is to depressurize the RCS to refill the Pressurizer" needs to be removed. This provides a very good cue to the applicants. The statement "In response to a small break LOCA" can be removed, fact that you are in ES-1.2 gives the same information. 3. The second part of the answer for choices C and D needs to be a full sentence like A and B.

4. Used on the Sequoyah 01/2009 retake exam.
5. Change Tave to Thot due to being in NC. 6. The second part of distracters A and B is not plausible. There is no cause and effect relationship between the cooldown and refilling the Pressurizer.

Q is U due to more than one non-plausible distracter.

04/18/2013 Facility Licensee modified Q as requested by NRC, and also modified the second part of distracters A and B. Q appears SAT at this time.

3 Q# 1. LOK (F/H) 2. LOD (1-5) 3. Psychometric Flaws

4. Job Content Flaws
5. Other 6. B/M/N 7. U/E/S 8. Explanation Stem Focus Cues T/F Cred. Dist. Partial Job-Link Minutia#/ units Back-ward Q= K/A SROOnly4 H 2 X Y N U K/A 011EG2.1.31 1. Q=K/A, Q=RO level
2. Overlaps with systems JPM B.
3. Distracter analysis does not match with initial conditions.
4. Question or JPM needs to be changed Q is U due to excessive overlap with op. test.

04/18/2013 Facility Licensee modified JPM to eliminate overlap as requested by NRC. Q appears SAT at this time.

05/20/2013

Minor typographical change provided to licensee for question statement. Q appears SAT at this time. 5 F 2 X Y N E K/A 015AG2.1.28 1. Q=K/A, Q=RO level 2. Need to choose different numbers for the second part of the question. AOP-R.04 step 2.2.1.b requires a reactor trip and RCP shutdown if leakoff flow is greater than 8 gpm. The current answer selected is equal to 8 gpm. It could easily be argued that the other distracter is correct. 3. Also need to specify whether or not the reactor trip would be an immediate manual since leakoff flow less than 8 gpm would lead to a shutdown or tripped reactor and securing of the RCP.

4. Overlaps with SRO question #86

04/18/2013 Facility Licensee modified Q as requested by NRC. Overlap will be assessed on the SRO question. Q appears SAT at this time.

05/20/2013

No overlap. Q appears SAT at this time.

4 Q# 1. LOK (F/H) 2. LOD (1-5) 3. Psychometric Flaws

4. Job Content Flaws
5. Other 6. B/M/N 7. U/E/S 8. Explanation Stem Focus Cues T/F Cred. Dist. Partial Job-Link Minutia#/ units Back-ward Q= K/A SROOnly6 H 2 X X Y N U K/A 022AK3.05 1. Q=K/A, Q=RO level 2. Choice A is not fully correct. Per AOP-M.09 a reactor trip is required if a reactor shutdown is required or desired. It does not specify for a power reduction. 3. Distracter B does not attempt to answer the question stated in the stem. It is a true/false statement. 4. Distracter D is not plausible because boration for the initial effects of Xenon only accounts for a power change in one direction.
5. Distracter C is not plausible.

Q is U due to more than one non-plausible distracter.

04/18/2013

Facility Licensee provided completely modified Q to address NRC concerns. Newly modified Q appears SAT at this time.

05/20/2013 Minor typographical changes provided to licensee for initial conditions. Q appears SAT at this time.

5 Q# 1. LOK (F/H) 2. LOD (1-5) 3. Psychometric Flaws

4. Job Content Flaws
5. Other 6. B/M/N 7. U/E/S 8. Explanation Stem Focus Cues T/F Cred. Dist. Partial Job-Link Minutia#/ units Back-ward Q= K/A SROOnly7 H 2 X Y B E K/A 025AK2.05 1. Q=K/A, Q=RO level 2. The word containment in the fourth bullet of the initial conditions is missing the letter "e".
3. Need to better specify what procedure is currently in effect, either directly or by providing more conditions. If already exited ES-1.3, EA-63-8 seeks TSC guidance prior to securing pumps.
4. Question used on Sequoyah 01/2008 exam.

04/18/2013

Facility Licensee modified Q as requested by NRC. Q appears SAT at this time.

5/09/2013

Facility requested to modify question following in-office review and validation. Additional comments below reflect the modified questions.

1. Q = K/A, Q = RO 2. Format 2 nd bullet in the initial conditions the same as the others 3. Second question statement should have is/a re to make all choices grammatically correct. 4. Need to be consisted regarding the nomenclature of containment spray pumps in the stem and answer choices (all Cntmt or all Containment) 5. Need to state in accordance with ES 1.3 for both question statements

05/20/2013 Licensee made necessary corrections. Q appears SAT at this time.

6 Q# 1. LOK (F/H) 2. LOD (1-5) 3. Psychometric Flaws

4. Job Content Flaws
5. Other 6. B/M/N 7. U/E/S 8. Explanation Stem Focus Cues T/F Cred. Dist. Partial Job-Link Minutia#/ units Back-ward Q= K/A SROOnly8 F 2 X Y N E K/A 026AA1.06 1. Q=K/A, Q=RO level 2. Plausibility for second part of choices C and D is weak. Why would an operator increase flow with no indication of temperature? Recommend simply asking if a flow rate adjustment is or is not required (not the direction of adjustment), or providing a plausible SFP temperature that is rising in the stem.

04/18/2013

Facility Licensee modified Q as requested by NRC (adjusted yes or no). Q appears SAT at this time.

5/20/2013

Need procedure reference for first part question statement (AOP M.03) and ask what is required to first be done vice what will be done.

05/21/2013 Licensee made required changes. Q appears SAT at this time. 9 H 2 X Y N U K/A 029EA1.09 1. Q=K/A, Q=RO level 2. Distracter C and D are not plausible with rod speed currently at 72 steps/min. Why would FR-S.1 direct lowering the speed of rod insertion?

Q is U due to more than one non-plausible distracter.

04/18/2013

Facility Licensee modified Q as requested by NRC. Q could be evaluated as an E due to ease of fix (change 72 spm to 'maximum rate'). Q appears SAT at this time. 10 H 2 X X Y N E K/A 040AA2.02 1. Q=K/A, Q=RO level 2. Need to be more specific in initial conditions and/or stem question to place applicant at a specific step in procedure. Per AOP-S.05, could trip due to personnel safety at 0701. Recommend providing a leak location.

3. Distracter analysis numbers do not match changes of MWe in table. 4. Based on a Tave/Tref calculation (using turbine power since Tref is not provided) the mismatch will be greater than 5 degrees prior to 0703.

04/18/2013

Facility Licensee modified Q as requested by NRC. Modified Q appears SAT at this time.

7 Q# 1. LOK (F/H) 2. LOD (1-5) 3. Psychometric Flaws

4. Job Content Flaws
5. Other 6. B/M/N 7. U/E/S 8. Explanation Stem Focus Cues T/F Cred. Dist. Partial Job-Link Minutia#/ units Back-ward Q= K/A SROOnly11 H 2 X X Y M E K/A 054AK1.01 1. Q=K/A, Q=RO level 2. Need to place the word "initiate" in front of "SI" in the stem question. 3. Recommend placing a value and increasing for containment pressure in the initial conditions since no indication is given for leak size and an initial value is given for hotwell level. Need to remove slowly from hotwell level decreasing in stem since this makes distracters A and B implausible. The actions stated in the stem question are not required by the AOP until containment pressure approaches 1.5 psig.

04/18/2013

Facility Licensee modified Q as requested by NRC. Q appears SAT at this time.

5/09/2013

Facility requested to modify question following in-office review and validation. Additional comments below reflect the modified questions.

1. Q = K/A, Q = RO 2. Need to remove "will" from the second question statement
3. Recommend using the word lower or decrease versus drop in the second question statement. 4. Recommend basing the question on the given conditions: "Based on the given conditions, which ONE of following completes the statements below?

5/20/2013 Licensee made necessary corrections. Q appears SAT at this time.

12 F 2 X X Y N E K/A 056G2.4.4 1. Original K/A was rejected and replaced. 2. Q=K/A, Q=RO level

3. Normal and abnormal are not very precise for the second choices. It could be argued that it is abnormal to have any diesels running. Recommend giving the last two bullets for initial conditions and just asking how many or what diesels should be operating. 4. Another option: (1) Entry conditions are met for the following procedure:______ (2) EDG status is/is not what is expected for the given plant conditions.

04/18/2013

Facility Licensee modified Q as requested by NRC. Q appears SAT at this time.

8 Q# 1. LOK (F/H) 2. LOD (1-5) 3. Psychometric Flaws

4. Job Content Flaws
5. Other 6. B/M/N 7. U/E/S 8. Explanation Stem Focus Cues T/F Cred. Dist. Partial Job-Link Minutia#/ units Back-ward Q= K/A SROOnly13 H 3 X X Y N S K/A 062AA2.01 1. Q=K/A, Q=RO level 2. Are there any sump high level alarms associated with the YARD AREA?

04/18/2013

Facility Licensee provided information as requested by NRC. Q appears SAT at this time w/no modification needed. 14 F 2 X Y B E K/A 065AK3.08 1. Q=K/A, Q=RO level

2. Q distracters contain 4 independent actions and 4 independent reasons. Due to K/A match concerns, recommend to change question to ask only reasons. Can cut out everything in the distracters before the word "to."

04/18/2013

Facility Licensee modified Q as requested by NRC. K/A is met because asking overall mitigative strategy implies the reason for taking actions. Q appears SAT at this time.

5/09/2013

Facility requested to modify question following in-office review and validation. Additional comments below reflect the modified questions.

1. Q = K/A, Q = RO
2. Distracter A is not plausible because it is how distracter B is accomplished (or vice versa). Another distracter needs to be selected that controls another pressure or flow. 3. Can you control S/G locally from the AUX Bldg? May or may not be plausible for that reason also. Maybe need to remove the AUX Bldg. from the stem question.

5/20/2013

Licensee made necessary corrections. Q appears SAT at this time.

15 F 2 S Y B S K/A 077AA1.03 1. Q=K/A, Q=RO level

2. Question used on Sequoyah 09/2010 exam. 3. Add statement in stem that everything is in auto/no equipment out of service.

04/18/2013 Facility Licensee requested to not modify the Q as requested by NRC. Chief Examiner agreed with the lack of change. Q appears SAT at this time.

9 Q# 1. LOK (F/H) 2. LOD (1-5) 3. Psychometric Flaws

4. Job Content Flaws
5. Other 6. B/M/N 7. U/E/S 8. Explanation Stem Focus Cues T/F Cred. Dist. Partial Job-Link Minutia#/ units Back-ward Q= K/A SROOnly16 F 2 X Y B E K/A W/E04EA2.2 1. Q=K/A, Q=RO level 2. Question used on Sequoyah 2010 exam.
3. To make second part of distracter A and C plausible (what if pzr level offscale low due to the LOCA?), add "or RVLIS level rising."

04/18/2013 Facility Licensee modified Q as requested by NRC. Q appears SAT at this time. 17 F 2 Y B E K/A W/E05EK2.2 1. Q=K/A, Q=RO level 2. Question used on Sequoyah 02/2010 exam 3. Every system listed in second part of distracters after the first system is unnecessary. Change second part of question to ask whether TDAFW pump or MDAFW pump would be used first.

04/18/2013 Facility Licensee modified Q as requested by NRC. Q appears SAT at this time. 18 H 2 X X Y N U K/A W/E11EK3.4 1. Q=K/A, Q=RO level 2. RO level because it is only asking major mitigation strategy. 3. The second part answers and distracters contain cues that can be derived from the stem or references. 4. The second part of distracters B and C is not plausible.

Q is U due to more than one non-plausible distracter.

04/18/2013

Facility Licensee modified Q as requested by NRC. Q appears SAT at this time. 19 H 2 X Y N E K/A 001AA2.05 1. Q=K/A, Q=RO level

2. Need quotes around all procedure titles (or none). Need to be consistent throughout the exam.

04/18/2013 Facility Licensee modified Q as requested by NRC. Q appears SAT at this time.

10 Q# 1. LOK (F/H) 2. LOD (1-5) 3. Psychometric Flaws

4. Job Content Flaws
5. Other 6. B/M/N 7. U/E/S 8. Explanation Stem Focus Cues T/F Cred. Dist. Partial Job-Link Minutia#/ units Back-ward Q= K/A SROOnly20 F 2 X Y B U K/A 036AK2.02 1. Q=K/A, Q=RO level 2. The answer is not totally correct. The AB is evacuated also IAW AOP-M.04. Also, per AOP-M.04 you check vice ensure containment isolation has actuated. 3. It is not plausible to evacuate only non-essential personnel or just the immediate or fuel handling area when the given radiation monitors are in alarm. Also, SFP and Rx Cavity levels have no effect on a dropped assembly.

Q is U due to more than one non-plausible distracter.

04/18/2013 Facility Licensee modified Q as requested by NRC and provided a new format to all distracters. Q appears SAT at this time.

5/09/2013

Facility requested to modify question following in-office review and validation. Additional comments below reflect the modified questions.

1. Q = K/A, Q = RO
2. Splitting out the questions is good.
3. Distracters A(1) and D(1) not plausible due to actually dropping a fuel assembly, all three of the monitors are in alarm and no other condition that would bring an automatic isolation into doubt.
4. Recommend basing the question on the given conditions: "Based on the given conditions, which ONE of following completes the statements below?
5. Evacuation on previous version is not SRO only. See note on page 12 of 42 of AOP-M.04. 6. Need to specify "manual isolation" in the second part question.

Question U due to two non plausible distracters.

5/20/2013

Licensee made necessary corrections. Q appears SAT at this time.

21 F 2 X Y N E K/A 037AG2.2.44 1. Q=K/A, Q=RO level 2. Need to state what step 1 of AOP-R.01 is in the initial conditions.

3. Need to indicate that normal letdown is in service in the initial conditions. 4. Need to put quotes around the procedure titles.

04/18/2013 Facility Licensee modified Q as requested by NRC. Q appears SAT at this time.

11 Q# 1. LOK (F/H) 2. LOD (1-5) 3. Psychometric Flaws

4. Job Content Flaws
5. Other 6. B/M/N 7. U/E/S 8. Explanation Stem Focus Cues T/F Cred. Dist. Partial Job-Link Minutia#/ units Back-ward Q= K/A SROOnly22 H 2 X Y N U K/A 051AA1.04 1. Q=K/A, Q=RO level 2. It is not plausible for generator output to rise if condenser pressure increases. This part is GFES. If testing the effect of turbine control being in IMPOUT, the choices should be lower or none. If applicant incorrectly believed that the turbine controls would adjust to maintain MWe constant, then the choice would be none. 3. Above the POAH with a given steam demand, rod motion maintains temperature vice power. 4. Recommend basing the stem question on the current/given conditions and using full sentences in the stem question.

Q is U due to more than one non-plausible distracter.

04/18/2013 Facility Licensee modified Q as requested by NRC. Still have issue with plausibility of distracter B at this time. Q was tabled, may need another K/A. Randomly selected a new K/A (032 AG 2.1.23). See question 22A. 22A H 2 X Y B U 05/01/2013

K/A 032 AG2.1.23 1. Q=K/A, Q= RO 2. Distracter A is not plausible. Why would you press reset and start on a SR Audio Count Rate Drawer when no SR instruments are energized.

3. Distracter B is not plausible because there is no switch identified as a source range manual block switch in any of the applicable procedures.
4. Choice C (correct answer) does not identify a switch nomenclature that is used in any of the applicable procedures for this situation. 5. See recommendation provided separately.

Question is U due to two non plausible distracters.

5/09/2013 Facility requested to modify question following in-office review and validation. Additional comments below reflect the modified questions.

1. Q = K/A, Q = RO
2. Distracters C and D are the same 3. Will discuss plausibility of distracters C(1) and D(1)

Question U due to two non plausible distracters 5/20/2013

Facility modified first part question and distracters provided by NRC. Q appears SAT at this time.

12 Q# 1. LOK (F/H) 2. LOD (1-5) 3. Psychometric Flaws

4. Job Content Flaws
5. Other 6. B/M/N 7. U/E/S 8. Explanation Stem Focus Cues T/F Cred. Dist. Partial Job-Link Minutia#/ units Back-ward Q= K/A SROOnly23 F 2 X Y M E K/A 059AK3.01 1. Original K/A was rejected and replaced. 2. Q=K/A
3. Do not believe the acronym "LCO" is applicable to the ODCM. 4. Question: Is this SRO level knowledge (bases of ODCM)? Question rated as "E" due to information being listed in lesson plan.

04/18/2013

Facility Licensee made slight modification as requested by NRC. Based on licensee input and lesson plan material, Q does not cover SRO-only information. Q appears SAT at this time. 24 H 3 Y N E K/A 074EA2.07 1. Q=K/A, Q=RO level 2. Modify stem to state that "all RCPs are stopped." 3. Modify second bullet to ensure that FRP monitoring is allowed. The team has made from E-0 to another procedure, etc.

04/18/2013

Facility Licensee modified Q as requested by NRC. Q appears SAT at this time. 25 H 2 X Y M U K/A W/E03EK1.1 1. Not plausible to stop the only running CCP.

2. Not plausible for SCM to remain the same with temperature rising even if pressure had also risen, it is not a linear relationship. 3. Choice B second part states, "the running CPP".
4. Remove bullet concerning small break LOCA from stem. 5. Put procedure title in quotes.

Q is U due to more than one non-plausible distracter.

04/18/2013

Facility Licensee modified Q as requested by NRC, second part distracter modified to choose between "establish normal charging" and "shutdown second SIP." Q appears SAT at this time. 26 H 2 X Y B E K/A W/E14EA1.1 1. Q=K/A, Q=RO level 2. Used on Sequoyah 0109 NRC exam.

3. Need to add opened from the MCR to the stem question. 4. Distracter analysis for choice A lists one incorrect valve number.

04/18/2013 Facility Licensee modified Q as requested by NRC. Q appears SAT at this time.

13 Q# 1. LOK (F/H) 2. LOD (1-5) 3. Psychometric Flaws

4. Job Content Flaws
5. Other 6. B/M/N 7. U/E/S 8. Explanation Stem Focus Cues T/F Cred. Dist. Partial Job-Link Minutia#/ units Back-ward Q= K/A SROOnly27 F 2 X Y B E K/A W/E15EK3.1 1. Q=K/A, Q=RO level 2. Used on Sequoyah 09/2010 exam
3. Need to remove "the containment sump" from the second part question in the stem. 4. Need to reduce second part answer and distracters to only ensure adequate SDM and determine level of activity.
5. Need to add Which ONE of the following to the beginning of the stem question.

04/18/2013

Facility Licensee modified Q as requested by NRC. Q appears SAT at this time.

5/09/2013

Facility requested to modify question following in-office review and validation. Additional comments below reflect the modified questions.

1. Q = K/A, Q = RO 2. Need to discuss what the two possibly correct answers were. It does not seem right that adding a true statement to the end of each choice would change anything. 3. The question choice formatting needs to be corrected. 4. Recommend basing the question on the given conditions: "Based on the given conditions, which ONE of following completes the statements below?

05/21/2013 Facility licensee made changes as required. Q appears SAT at this time.

28 H 2 Y B S K/A 003K6.02 1. Q=K/A, Q=RO level

Q appears SAT at this time. 29 H 2 x x Y B E K/A 004A2.06 1. Q=K/A, Q=RO level

2. Question used on Watts Bar 10/2011exam. 3. Need to ask the action the RO is required to take in the stem question, 4. Need to add that the instantaneous thermal power (U1118) is below applicable limits to the stem conditions. 5. For second part of B and D, replace initiate normal boration with perform RCS boration using O-SO-62-7. (more closely match procedural action format) 04/18/2013

Facility Licensee modified Q as requested by NRC. Q appears SAT at this time.

14 Q# 1. LOK (F/H) 2. LOD (1-5) 3. Psychometric Flaws

4. Job Content Flaws
5. Other 6. B/M/N 7. U/E/S 8. Explanation Stem Focus Cues T/F Cred. Dist. Partial Job-Link Minutia#/ units Back-ward Q= K/A SROOnly30 F 2 Y N E K/A 004A.4.12 1. Q=K/A, Q=RO level 2. Add that the dilution has terminated automatically to the stem.
3. Be clear at which point in the procedural restoration lineup the question is asking.

04/18/2013

Facility Licensee modified Q as requested by NRC. Q appears SAT at this time. 31 H 2 N B U K/A 005A1.03 1. Question does not match the K/A. The question does not address closed cooling water (CCS) flow rate or temperature in any way.

Q is U due to not meeting the K/A

04/18/2013

Facility Licensee provided a modified Q to address concerns raised by NRC. Q appears SAT at this time.

5/09/2013

Facility requested to modify question following in-office review and validation. Additional comments below reflect the modified questions.

1. Q = K/A, Q = RO 2. Need to correct question choice formatting
3. Possibly no correct answer. Per GO-1 the heat exchanger outlet valve is also throttled and a note states that CCS may be throttled. Per RHR lesson plan, these RHR butterfly valves have 500 gpm leak by and CCS may have to be throttled. May need to

define the specific time per iod in the stem question. 4. Recommend basing the question on the given conditions: "Based on the given conditions, which ONE of following completes the statements below?

05/21/2013 Facility licensee made changes as required. Q appears SAT at this time.

15 Q# 1. LOK (F/H) 2. LOD (1-5) 3. Psychometric Flaws

4. Job Content Flaws
5. Other 6. B/M/N 7. U/E/S 8. Explanation Stem Focus Cues T/F Cred. Dist. Partial Job-Link Minutia#/ units Back-ward Q= K/A SROOnly32 H 2 X Y B E K/A 006K5.05 1. Q=K/A, Q=RO level 2. Used on CPNPP 03/2010 NRC exam.
3. Choice A is also correct and needs to be changed. 4. Replace "affect" in stem with "effect."

04/18/2013 Facility Licensee modified Q as requested by NRC. Q appears SAT at this time.

5/09/2013 Facility requested to modify question following in-office review and validation. Additional comments below reflect the modified questions.

1. Distracter D is not plausible.
2. Distracter C is a subset of Distracter D. 3. E-0 does not identify the adverse effect anywhere, it just tells you what to do. 4. If it is identified in EPM-3-E-0, then state that vice E-0 in the stem question.
5. EPM-3-E-0 discusses filling the pressurizer not pressurizing to the PORV setpoint. 6. It is not RO knowledge to be able to cross reference one statement in the EPM to another statement in the WOG Generic Executive Volume.
7. Distracter B is not plausible.

Question U due to two non plausible distracters and license level mismatch as written.

05/21/2013

Facility licensee made changes as required. Q appears SAT at this time.

33 F 2 Y B S K/A 007A3.01 1. Q=K/A, Q=RO level 2. Used on the Watts Bar 06/2011 exam.

Q appears SAT at this time.

16 Q# 1. LOK (F/H) 2. LOD (1-5) 3. Psychometric Flaws

4. Job Content Flaws
5. Other 6. B/M/N 7. U/E/S 8. Explanation Stem Focus Cues T/F Cred. Dist. Partial Job-Link Minutia#/ units Back-ward Q= K/A SROOnly34 H 2 X X Y B E K/A 007G2.1.20 1. Q=K/A, Q=RO level 2. Choice C is partially correct. If temperature did not reduce to less than 120 degrees prior to level reaching 88%, you would stop there then drain some and fill again. An applicant may discount choice D because of knowing you would stop at 88% if the desired reduction in temperature had not yet occurred. Need to tighten up the answer and distracter.

04/18/2013

Facility Licensee modified Q as requested by NRC. Removed setpoints from the various distracters, opened up the second part Q. Q appears SAT at this time. 35 F 2 X Y B E K/A 008A3.01 1. Original K/A was rejected and replaced. 2. Q=K/A, Q=RO level 3. The distracters with miniflow valve open are not plausible, because these pumps do not have them. Recommend just removing that portion of the distracter. 4. Remove periods at end of answer and distracters.

04/18/2013

Facility Licensee modified Q as requested by NRC. Q appears SAT at this time. 36 H 2 X Y N E K/A 010K6.01 1. Q=K/A, Q=RO level 2. Recommend adding to the first part question statement: Before any operator actions occur, the failure ____ result in the Pressurizer backup heaters being automatically energized.

04/18/2013

Facility Licensee modified Q as requested by NRC. Q appears SAT at this time. 37 H 2 X Y B E K/A 010K6.03 1. Q=K/A, Q=RO level

2. Used on the Sequoyah 1/2008 NRC exam 3. Remove assume from stem, replace with if. Remove capitalization from no operator actions.
4. For second part question, just ask if reactor automatically trips or does not automatically trip.

04/18/2013 Facility Licensee modified Q as requested by NRC. Q appears SAT at this time.

17 Q# 1. LOK (F/H) 2. LOD (1-5) 3. Psychometric Flaws

4. Job Content Flaws
5. Other 6. B/M/N 7. U/E/S 8. Explanation Stem Focus Cues T/F Cred. Dist. Partial Job-Link Minutia#/ units Back-ward Q= K/A SROOnly38 F 2 Y B E K/A 012K2.01 1. Q=K/A, Q=RO level 2. Used on the Watts Bar 10/2011 NRC exam.
3. Distracter analysis for choice A has does not indicate that the answer is correct. 4. Question: Is there a difference between a small v and capital V for voltage? Just needs to be a consistent format throughout the exam.

04/18/2013

Facility Licensee modified Q as requested by NRC. Q appears SAT at this time.

5/09/2013

Facility requested to modify question following in-office review and validation. Additional comments below reflect the modified questions.

1. Q = K/A, Q = RO 05/21/2013

Facility licensee made changes as required. Q appears SAT at this time.

39 H 3 X Y B E K/A 013K4.07 1. Q=K/A, Q=RO level 2. Do not believe it is plausible for both trains of slave relays (ECCS equipment) to actuate when only one train of master relays actuate (choice C). 3. Recommend for stem question: If no operator actions, which ONE of the following identifies how SSPS and ECCS will automatically respond?

04/18/2013

Facility Licensee modified Q as requested by NRC. Q appears SAT at this time. 40 F 2 Y B S K/A 022K4.03 1. Q=K/A, Q=RO level 2. Used on Sequoyah 1/2009 NRC exam.

3. Reviewed lesson plan. There appears to be enough differences in the systems to make choices A and B acceptable as separate distracters.

04/18/2013 Q appears SAT at this time.

18 Q# 1. LOK (F/H) 2. LOD (1-5) 3. Psychometric Flaws

4. Job Content Flaws
5. Other 6. B/M/N 7. U/E/S 8. Explanation Stem Focus Cues T/F Cred. Dist. Partial Job-Link Minutia#/ units Back-ward Q= K/A SROOnly41 H 3 Y M E K/A 025A4.02 1. Q=K/A, Q=RO level 2. Recommend only asking if red light is lit or not for second part of question.
3. Recommend using Phase A isolation for distracters on first part choices A and B.

04/18/2013

Facility Licensee modified Q as requested by NRC. Q appears SAT at this time. 42 F 2 X X Y B U K/A 026A1.02 1. Original K/A was rejected and replaced.

2. Q=K/A, SRO level. 3. 125 °F is not plausible for a containment design temperature. 4. Second part of stem question needs wording at the end corrected.
5. Recommend testing the above the line TS containment temperature requirements for upper and lower containment for the first part question (105 degrees and 125 degrees).

This is above the line TS information.

Q is U due to more than one non-plausible distracter and license level mismatch.

04/18/2013

Facility Licensee modified Q as requested by NRC. Q appears SAT at this time.

5/09/2013 Facility requested to modify question following in-office review and validation. Additional comments below reflect the modified questions.

1. Q = K/A, Q = RO
2. Need to identify the Tech Spec in the first question statement (3.6.1.5 --) 3. Based on the distracter analysis provided, B is not the correct answer. The distracter analysis is incorrect.
4. Recommend basing the question on the given conditions: "Based on the given conditions, which ONE of following completes the statements below?
5. Need to reformat the first bullet 05/21/2013

Requested that the licensee add the applicable TS name to the first question statement. Facility licensee made changes as requested. Q appears SAT at this time.

19 Q# 1. LOK (F/H) 2. LOD (1-5) 3. Psychometric Flaws

4. Job Content Flaws
5. Other 6. B/M/N 7. U/E/S 8. Explanation Stem Focus Cues T/F Cred. Dist. Partial Job-Link Minutia#/ units Back-ward Q= K/A SROOnly43 F 2 X Y B E K/A 026K1.01 1. Q=K/A, Q=RO level 2. Question used on Watts Bar 2006 NRC exam.
3. Choice C is not plausible. It is common knowledge that operating a pump with the suction valve closed will damage the pump. 4. Recommend adding to the stem statement that the valve cannot be opened using the switch in the MCR.

04/18/2013

Facility Licensee modified Q as requested by NRC. Q appears SAT at this time. 44 H 3 Y B S K/A 039K3.06 1. Q=K/A, Q=RO level

2. Question used on the Watts Bar 06/2011 NRC exam.

04/18/2013

Q appears SAT at this time. 45 H 2 X X Y B U K/A 039K5.08 1. Q=K/A, Q=RO level

2. Choice D distracter analysis states that the MSIVs would eventually close to stop the cool down. If this led to a reactor trip it would be a negative reactivity addition. Need to tighten up the stem question to specify initial effect, etc. Need to specify timing.

Q is U due to more than one non-plausible distracter.

04/18/2013 Facility Licensee modified Q as requested by NRC. Q appears SAT at this time. 46 H 3 X Y B U K/A 059A3.04 1. Q=K/A, Q=RO level 2. The distracter analysis for choice A does not make sense. Why would it be plausible to think that you would need to increase output if steam pressure was less? Whether median or average, when the highest number is failed the indicated steam pressure would decrease. 3. Based on choice C distracter analysis, this could possibly be another correct answer or is not plausible. 4. Based on 1-SO-98-1, it would transfer to manual if two of the three instruments failed. If a transfer to manual occurred, would the output remain the same?

Q is U due to more than one non-plausible distracter.

04/18/2013 Facility Licensee presented modified Q as requested by NRC with four new distracters (version 546). Q appears SAT at this time.

20 Q# 1. LOK (F/H) 2. LOD (1-5) 3. Psychometric Flaws

4. Job Content Flaws
5. Other 6. B/M/N 7. U/E/S 8. Explanation Stem Focus Cues T/F Cred. Dist. Partial Job-Link Minutia#/ units Back-ward Q= K/A SROOnly47 F 2 Y N S K/A 061K2.01 1. Q=K/A, Q=RO level 2. Use upper or lower case V for voltage consistently throughout the exam. 48 H 2 X Y B E K/A 062A2.09 1. Q=K/A, Q=RO level 2. Add a stem bullet that the unit remains at power. 3. Recommend for second part question: The appropriate procedure does or does not require a manual reactor trip. 4. Need to remove the "s" from recloses in choice C and D.

5/09/2013 Facility requested to modify question following in-office review and validation. Additional comments below reflect the modified questions.

1. Q = K/A, Q = RO
2. Recommend basing the question on the given conditions: "Based on the given conditions, which ONE of following completes the statements below?
3. "Operation of the overcurrent relay will cause" does not need to be in the first question statement since it is stated in the initial conditions 4. Recommend adding "automatically" to the first question statement.

05/21/2013 Facility licensee made changes as required. Q appears SAT at this time.

49 F 2 X X Y B U K/A 063A1.01 1. Q=K/A, SRO level (coping time/FSAR basis)

2. Choices A and B are not plausible, stripping loads are a major mitigation strategy for ECA 0.0. 3. Recommend asking time for stripping loads for 125 VDC batteries and DC air-side seal oil pump breakers.
4. Overlap with SRO #99.

Q is U due to more than one non-plausible distracter and license level mismatch.

04/18/2013

Facility Licensee modified Q as requested by NRC. Q appears SAT at this time.

21 Q# 1. LOK (F/H) 2. LOD (1-5) 3. Psychometric Flaws

4. Job Content Flaws
5. Other 6. B/M/N 7. U/E/S 8. Explanation Stem Focus Cues T/F Cred. Dist. Partial Job-Link Minutia#/ units Back-ward Q= K/A SROOnly50 H 3 X Y B E K/A 063K3.02 1. Q=K/A, Q=RO level 2. Question used on the Watts Bar 10/2011 exam.
3. Recommend adding information in the stem indicating that a blackout has not occurred. 4. Need to capitalize the S, I in safety injection.

04/18/2013

Facility Licensee modified Q as requested by NRC. Q appears SAT at this time. 51 H 3 X Y N U K/A 064K3.03 1. Q=K/A, Q=RO level 2. Choice A and D not plausible: if MW control works, why would VAR control not work? also if MW control does not work, why would VAR control work? cause/effect relationship?

Q is U due to more than one non-plausible distracter.

04/18/2013

Facility Licensee presented a fully modified Q to incorporate NRC comments. Q appears SAT at this time.

5/09/2013 Facility requested to modify question following in-office review and validation. Additional comments below reflect the modified questions.

1. Q = K/A, Q = RO
2. With changing the SDBD that has the loss of voltage to the opposite train on the other unit, this makes this a simple unit and train separation issue 3. LOD = 1
4. Recommend basing the question on the given conditions: "Based on the given conditions, which ONE of following completes the statements below?
5. Need to capitalize "one" in the question statement to be consistent.

Question U due to two non plausible distracters and LOD = 1.

05/21/2013

Facility licensee made changes as required. Q appears SAT at this time.

22 Q# 1. LOK (F/H) 2. LOD (1-5) 3. Psychometric Flaws

4. Job Content Flaws
5. Other 6. B/M/N 7. U/E/S 8. Explanation Stem Focus Cues T/F Cred. Dist. Partial Job-Link Minutia#/ units Back-ward Q= K/A SROOnly52 H 2 X Y B U K/A 073K1.01 1. Q=K/A, Q=RO level 2. Used on the Sequoyah 09/2010 NRC exam.
3. A design resulting in a negative pressure during a high radiation condition is not plausible. The control building ventilation lesson plan states that the system maintains a positive pressure at all times except during a tornado. It does not seem plausible that an Emergency Air Pressurization fan would create a negative pressure in the space it is discharging to.

Q is U due to more than one non-plausible distracter.

04/18/2013

Facility Licensee modified Q as requested by NRC. Q appears SAT at this time. 53 H 2 X X Y N E K/A 076A4.02 1. Q=K/A, Q=RO level

2. Need to discuss if knowledge of whether the valve opens at 40 vice 200 rpm is minutia since the D/G immediately accelerates to 400 rpm. The procedure does not discuss valve position versus rpm.
3. Replace ERCW valve with the specific valve nomenclature in second part question. 4. JPM I is an alternate path JPM where this valve fails to open. The procedure that is given for the JPM does not specify the rpm value or shutting down the diesel. 5. Include procedure reference for "normal start".

04/18/2013

Facility Licensee modified Q as requested by NRC. Q appears SAT at this time.

05/21/2013

Requested minor editorial change on valve nomenclature. Facility licensee made changes as requested. Q appears SAT at this time.

23 Q# 1. LOK (F/H) 2. LOD (1-5) 3. Psychometric Flaws

4. Job Content Flaws
5. Other 6. B/M/N 7. U/E/S 8. Explanation Stem Focus Cues T/F Cred. Dist. Partial Job-Link Minutia#/ units Back-ward Q= K/A SROOnly54 F 2 X Y N U K/A 078G2.4.35 1. Q=K/A, Q=RO level 2. Choices A and B do not appear to be plausible. When does a field operator have the capability to reset a phase B signal? Also, nothing in the stem deals with starting or stopping compressors.

Q is U due to more than one non-plausible distracter.

04/18/2013

Facility Licensee modified Q as requested by NRC. Q appears SAT at this time.

5/09/2013

Facility requested to modify question following in-office review and validation. Additional comments below reflect the modified questions.

1. Q = K/A, Q = RO 2. Recommend basing the question on the given conditions: "Based on the given conditions, which ONE of following completes the statements below?

05/21/2013 Facility licensee made changes as required. Q appears SAT at this time.

24 Q# 1. LOK (F/H) 2. LOD (1-5) 3. Psychometric Flaws

4. Job Content Flaws
5. Other 6. B/M/N 7. U/E/S 8. Explanation Stem Focus Cues T/F Cred. Dist. Partial Job-Link Minutia#/ units Back-ward Q= K/A SROOnly55 F 2 X Y N E K/A 103K1.01 1. Q=K/A, Q=RO level 2. Based on the A-A cooler being preferred and that the PZR enclosure may heat up if the other three were operating; then even if the A-A tripped and another one was in A-P Auto and auto started, the PZR enclosure would still heat up. This could render the second part of distracters C and D implausible.
3. Recommend asking if the standby cooler would auto start or not. Could specify the switch position.

04/18/2013

Facility Licensee modified Q as requested by NRC. Q appears SAT at this time.

5/09/2013 Facility requested to modify question following in-office review and validation. Additional comments below reflect the modified questions.

1. Q = K/A, Q = RO
2. Recommend basing the question on the given conditions: "Based on the given conditions, which ONE of following completes the statements below?
3. Based on logic and knowing that one unit has 2 UCCUs and the other has 4 UCCUs and given the fact that two of them are running in the initial conditions, distracters A(2) and B(2) are not plausible. You could never say that there were or were not any

additional coolers available to place in service on both units. 4. Question: Is it possible that a loss of all cooling in the lower compartment would not eventually cause an increase in temperature in the upper compartment? This may be a timing issue.

Question U due to two non plausible distracters

05/21/2013

Facility licensee made changes as required. Q appears SAT at this time.

25 Q# 1. LOK (F/H) 2. LOD (1-5) 3. Psychometric Flaws

4. Job Content Flaws
5. Other 6. B/M/N 7. U/E/S 8. Explanation Stem Focus Cues T/F Cred. Dist. Partial Job-Link Minutia#/ units Back-ward Q= K/A SROOnly56 F 2 Y N E K/A 002K5.07 1. Q=K/A, Q=RO level 2. In GO-2 step 5.3.[9], you will borate or deborate if ECB does not approximately equal the actual boron concentration. What is the definition of approximately? Need to clarify or this could lead to no correct answer.

04/18/2013 Facility Licensee modified Q as requested by NRC. Q appears SAT at this time.

05/21/2013 Requested minor editorial change on stem statement. Facility licensee made changes as requested. Q appears SAT at this time.

57 F 2 Y N E K/A 015A1.02 1. Q=K/A, Q=RO level 2. Recommend adding a power level to the stem (5 X 10 -4% and rising).

04/18/2013 Facility Licensee modified Q as requested by NRC. Q appears SAT at this time.

05/21/2013

Requested minor editorial change on stem statement. Facility licensee made changes as requested. Q appears SAT at this time.

26 Q# 1. LOK (F/H) 2. LOD (1-5) 3. Psychometric Flaws

4. Job Content Flaws
5. Other 6. B/M/N 7. U/E/S 8. Explanation Stem Focus Cues T/F Cred. Dist. Partial Job-Link Minutia#/ units Back-ward Q= K/A SROOnly58 H 3 X Y B U K/A 016A3.01 1. Q=K/A, Q=RO level 2. Question used on the Watts Bar 06/2011 NRC exam
3. Due to the first part of the stem question noting a larger affect, choices B and D are eliminated as plausible choices.

Q is U due to more than one non-plausible distracter.

04/18/2013

Facility Licensee modified Q as requested by NRC. Q appears SAT at this time.

5/09/2013

Facility requested to modify question following in-office review and validation. Additional comments below reflect the modified questions.

1. Q = K/A, Q = RO 2. Recommend basing the question on the given conditions: "Based on the given conditions, which ONE of following completes the statements below? 3. There could be a time issue with this question based on overshoot of level. Recommend adding bullets to the initial conditions that state that charging is in manual and that the failure of a temperature instrument results in a 5% difference between program and actual pressurizer level. Then ask "Based on the given conditions, if no operator actions occurred, WOOTF completes the statements below?
1. The instrument that failed is the ________ .
2. The pressurizer backup heaters ________ be automatically energized.

05/21/2013 Facility licensee made changes as required. Q appears SAT at this time.

27 Q# 1. LOK (F/H) 2. LOD (1-5) 3. Psychometric Flaws

4. Job Content Flaws
5. Other 6. B/M/N 7. U/E/S 8. Explanation Stem Focus Cues T/F Cred. Dist. Partial Job-Link Minutia#/ units Back-ward Q= K/A SROOnly 59 F 1 X Y B U K/A 027K2.01 1. Q=K/A, Q=RO level 2. LOD=1 due to being entirely a train question. How could distracters A, D be plausible even if cross train/cross unit power supplies?

Q is U due to more than one non-plausible distracter, LOD=1.

04/18/2013

Facility Licensee modified Q as requested by NRC. Q appears SAT at this time.

05/21/2013

Requested minor editorial change on stem statement. Facility licensee made changes as requested. Q appears SAT at this time.

60 F 2 Y N S K/A 033A2.01 1. Q=K/A, Q=RO level.

Q appears SAT at this time. 61 H 2 X Y B E K/A 034K6.02 1. Q=K/A, Q=RO level 2. Need to specify which technical specification in the stem question. 3. Used on the Sequoyah 1/2009 NRC retake exam.

04/18/2013

Facility Licensee modified Q as requested by NRC. Q appears SAT at this time. 62 H 2 X Y B E K/A 071K3.05 1. Q=K/A, Q=RO level

2. Not plausible for the waste gas radiation monitor to detect the release when it is leaking outside of the tank and there is also no release in progress.
3. What is the document/basis/lesson plan can be used to ensure choice A is the one and only correct answer?

04/18/2013

Facility Licensee significantly modified Q as requested by NRC. Q appears SAT at this time.

28 Q# 1. LOK (F/H) 2. LOD (1-5) 3. Psychometric Flaws

4. Job Content Flaws
5. Other 6. B/M/N 7. U/E/S 8. Explanation Stem Focus Cues T/F Cred. Dist. Partial Job-Link Minutia#/ units Back-ward Q= K/A SROOnly63 F 2 X Y N U K/A 072A4.01 1. Q=K/A, Q=RO level 2. Distracters B(2) and D(2) are not plausible for an area radiation monitor, ARMs do not have any sample flow.

Q is U due to more than one non-plausible distracter.

04/18/2013

Facility Licensee modified Q as requested by NRC. Q appears SAT at this time. 64 F 2 X Y N E K/A 079K4.01 1. Q=K/A, Q=RO level 2. Need to remove the extra wording for choices A(1) and C(1) to make the choices symmetrical. This is extra informat ion and provides teaching in the stem. 3. Need to add wording in the second part of the stem questions that specifies if the valve has closed.

04/18/2013

Facility Licensee modified Q as requested by NRC. Q appears SAT at this time. 65 F 2 Y N S K/A 086K1.03 1. Q=K/A, Q=RO level 2. Add the procedure reference to the question statement.

Q appears SAT at this time. 66 F 2 X X Y B E K/A G2.1.15 1. Q=K/A, Question appears SRO level

2. Need to see reference ODM-Y? 3. The stem question needs to be worded to form a complete sentence.

Q is U due to license level mismatch.

04/18/2013

Facility Licensee modified Q as requested by NRC. Q appears SAT at this time.

05/21/2013 Requested minor editorial change on stem statement. Facility licensee made changes as requested. Q appears SAT at this time.

29 Q# 1. LOK (F/H) 2. LOD (1-5) 3. Psychometric Flaws

4. Job Content Flaws
5. Other 6. B/M/N 7. U/E/S 8. Explanation Stem Focus Cues T/F Cred. Dist. Partial Job-Link Minutia#/ units Back-ward Q= K/A SROOnly67 H 4 X Y B U K/A G2.1.43 1. Q=K/A, Q=RO level 2. The title of the procedure makes the first part of C and D very weakly plausible. 3. The second part of distracter A and C are not plausible. How does rate of power change affect MTC?

Q is U due to more than one non-plausible distracter.

04/18/2013

Facility Licensee modified Q as requested by NRC. Q appears SAT at this time. 68 H 2 X Y B E K/A G2.2.22 1. Q=K/A, Q=RO level

2. Need to adjust the stem question to allow for the multiple number distracters to be grammatically correct if placed in the sentence.

04/18/2013 Facility Licensee modified Q as requested by NRC. Q appears SAT at this time. 69 H 2 Y B S K/A G2.2.42 1. Q=K/A, Q=RO level Q appears SAT at this time.

30 Q# 1. LOK (F/H) 2. LOD (1-5) 3. Psychometric Flaws

4. Job Content Flaws
5. Other 6. B/M/N 7. U/E/S 8. Explanation Stem Focus Cues T/F Cred. Dist. Partial Job-Link Minutia#/ units Back-ward Q= K/A SROOnly70 H 2 Y N E K/A G2.2.43 1. Q=K/A, Q=RO level 2. Recommend modifications to the Q statement and distracters to reduce the Q to the logical minimums as follows: (1) In accordance with OPDP-4, "Annunciator Disablement," the MINIMUM level of authorization required to disable the nuisance alarm ___(1)___ the Unit Supervisor. (2) In accordance with OPDP-4, it ___(2)___ required to make an entry regarding the annunciator disablement in the narrative log.

(1) is/is NOT (2) is/is NOT 04/18/2013

Facility Licensee modified Q as requested by NRC. Q appears SAT at this time.

5/09/2013 Facility requested to modify question following in-office review and validation. Additional comments below reflect the modified questions.

1. New question may not be RO level. Declaring an alarm a nuisance alarm appears to be more of an SRO duty per OPDP-4. A lot of evaluation seems to be required. 2. Answer selected for previous question may have been incorrect. Per OPDP-4 the US or SM may sign the sheet, but another portion seems to indicate that SM approval may be required. 3. Recommend asking how disabled annunciator is identified at Sequoyah for one of the questions. Blue dot or orange sticker on window, etc.

05/21/2013

Q appears SAT at this time.

71 H 1 X Y B U K/A G2.3.11 1. Q=K/A, Q=RO level

2. Distracters not plausible, LOD = 1. Only need to ID ruptured S/G. 3. Need to understand where in the procedure this question is asked to ensure 'B' is the one and only one correct answer; if the operators were in E-3 and had already set the atmospheric valve to the higher setpoint, would they continue to adjust the setpoint higher?

Q is U due to more than one non-plausible distracter.

04/18/2013

Facility Licensee modified Q as requested by NRC. Q appears SAT at this time.

31 Q# 1. LOK (F/H) 2. LOD (1-5) 3. Psychometric Flaws

4. Job Content Flaws
5. Other 6. B/M/N 7. U/E/S 8. Explanation Stem Focus Cues T/F Cred. Dist. Partial Job-Link Minutia#/ units Back-ward Q= K/A SROOnly72 F 2 X Y B E K/A G2.3.5 1. Q=K/A, Q=RO level 2. Recommend splitting stem question into two separate questions. 3. Used on the Sequoyah 2/2010 NRC exam.

04/18/2013

Facility Licensee modified Q as requested by NRC. Q appears SAT at this time. 73 F 2 X X Y B E K/A G2.4.8 1. Original K/A was rejected and replaced.

2. Q=K/A, Q=RO level 3. Used on the Sequoyah 01/2009 exam. 4. Distracter 'A' is a potential correct answer. First part of distracters 'C' and 'D' would need to be enhanced to specify which procedure the team is going to enter. 5. Because the second part distracters ar e all unique choices, can eliminate the need for the first part question. Better way to ask may be to use specific examples from EPM-4 3.11.7.B asking what are requirements to perform the AOP in ES-0.1?

04/18/2013

Facility Licensee modified Q as requested by NRC. Q appears SAT at this time.

05/21/2013

Identified during final review that distracter D(2) did not correlate to the stem question. Contacted licensee to make corrections.

32 Q# 1. LOK (F/H) 2. LOD (1-5) 3. Psychometric Flaws

4. Job Content Flaws
5. Other 6. B/M/N 7. U/E/S 8. Explanation Stem Focus Cues T/F Cred. Dist. Partial Job-Link Minutia#/ units Back-ward Q= K/A SROOnly74 H 2 X X Y N E K/A G2.4.3 1. Q=K/A, Q=RO level 2. Potential for logical elimination of distracters 'A' and 'C' based upon differences between the pictures' labels and nomenclature. 3. Need to make the picture of 1-PR-1-2 bigger and clearer; it is hard to recognize the boxed "C2" in the corner. Putting the picture on another page is fine.
4. Recommendation: given the change in 3. above, replace the distracters with the following: A. 1-PI-1-2A is a PAM indication. 1-PR-1-2 is NOT a PAM indication. B. 1-PI-1-2A is NOT a PAM indication. 1-PR-1-2 is a PAM indication. C. Both 1-PI-1-2A AND 1-PR-1-2 are PAM indications. D. Neither 1-PI-1-2A NOR 1-PR-1-2 are PAM indications.

04/18/2013 Facility Licensee modified Q as requested by NRC. Q appears SAT at this time.

5/09/2013

Facility requested to modify question following in-office review and validation. Additional comments below reflect the modified questions.

1. Knowledge of FSAR is generally more of an SRO level requirement. 2. Recommend tying question to EPM-4 Section 3.6.2. This is more of an RO requirement (instruments to using while performing EOP actions).

05/21/2013

Facility licensee made changes as required. Q appears SAT at this time.

75 H 2 X Y B E K/A G2.4.9 1. Q=K/A, Q=RO level 2. Need to add to the initial conditions in the stem that one CCP is operating.

04/18/2013 Facility Licensee modified Q as requested by NRC. Q appears SAT at this time.

SRO Questions

33 Q# 1. LOK (F/H) 2. LOD (1-5) 3. Psychometric Flaws

4. Job Content Flaws
5. Other 6. B/M/N 7. U/E/S 8. Explanation Stem Focus Cues T/F Cred. Dist. Partial Job-Link Minutia#/ units Back-ward Q= K/A SROOnly76 H 2 X X Y N N U K/A 007EA2.02 1. Q=K/A, Question is weak SRO only 2. Question needs to ask what procedure the crew is required to implement. 3. Second part of question is RO level. 4. Distracters A(1) and B(1) are not plausible because yellow path entry is optional. 5. Potential fix: use EA-3-8 procedure for first part distracters 'A' and 'B'?

Q is U due to more than one non-plausible distracter.

04/18/2013

Facility Licensee modified Q as requested by NRC. Q appears SAT at this time.

34 Q# 1. LOK (F/H) 2. LOD (1-5) 3. Psychometric Flaws

4. Job Content Flaws
5. Other 6. B/M/N 7. U/E/S 8. Explanation Stem Focus Cues T/F Cred. Dist. Partial Job-Link Minutia#/ units Back-ward Q= K/A SROOnly77 H 3 X X Y Y N E K/A 011EA2.08 1. Q=K/A, Q= SRO only
2. Distracter D is non plausible because no time is given post-LOCA. Distracter A is weakly plausible because subcooling is zero. 3. Remove "slowly" from trends in stem conditions. 4. Q, with above recommendations:

"Given the following plant conditions:

-Unit 2 was operating at 100% power when a LOCA occurred four (4) hours ago.

-The operating crew is performing E-1, "Loss of Reactor or Secondary Coolant," and has reached the step to "MONITOR SI termination criteria."

-Current conditions are: Core Exit T/Cs - 220 °F and stable RCS pressure - 16 psig and stable PZR level - 0% RVLIS Lower Range - 43% and trending up Containment pressure - 5.7 psig and trending down All 4 SG NR levels - 32-34% and stable SG 2, 3, & 4 pressures - 880 psig and stable SG 1 pressure - 820 psig and trending down RWST level - 26% and trending down Which ONE of the following completes the below statement?

Based on the above conditions, the Unit Supervisor is required to transition to ____________ . A. ES-1.1, "SI Termination" B. E-2, "Faulted Steam Generator" C. ES-1.3, "Transfer to RHR Containment Sump" D. ES-1.4, "Transfer to Hot Leg Recirculation" 5/09/2013

Facility requested to modify question following in-office review and validation. Additional comments below reflect the modified questions.

1. Q = K/A, Q = SRO only 2. The time for the second bullet needs to be 1200. 3. Need to remove reference to fold out page for choice B.

Requested minor editorial change on stem statement. Facility licensee made changes as requested. Q appears SAT at this time.

05/21/2013 Requested RCS pressure in initial conditions be changed back to 10# as previously discussed. Facility licensee made changes as requested. Q appears SAT at this time.

35 Q# 1. LOK (F/H) 2. LOD (1-5) 3. Psychometric Flaws

4. Job Content Flaws
5. Other 6. B/M/N 7. U/E/S 8. Explanation Stem Focus Cues T/F Cred. Dist. Partial Job-Link Minutia#/ units Back-ward Q= K/A SROOnly78 H 3 X Y Y N E K/A 025AA2.05 1. Q=K/A, Q= SRO only 2. Minor editorial, change "-and the crew will ___(2)___" to "-and the crew is required to ___(2)___".

04/18/2013

Facility Licensee modified Q as requested by NRC. Q appears SAT at this time. 79 H 2 Y N N U K/A 026AG2.4.45 1. Q=K/A, Question is not SRO only

2. Question can be answered solely using RO systems knowledge. RO knowledge to know first listed annunciator is CCW system related, off the Train A header. There are no other annunciators or radiation monitors listed in alarm that would imply an RCS (or charging/letdown) system issue.

Q is U due to license level mismatch.

04/18/2013

Facility Licensee modified Q as requested by NRC. Q appears SAT at this time.

36 Q# 1. LOK (F/H) 2. LOD (1-5) 3. Psychometric Flaws

4. Job Content Flaws
5. Other 6. B/M/N 7. U/E/S 8. Explanation Stem Focus Cues T/F Cred. Dist. Partial Job-Link Minutia#/ units Back-ward Q= K/A SROOnly80 H 3 X X Y Y N U K/A 056AG2.2.40 1. Q=K/A, Q= SRO only 2. The question appears to be asking the maximum amount of time that can be taken to get outside of the mode of applicability for the given conditions without restoring compliance.
3. Should give the applicable pages from Unit 2 TS book since the question is being asked for Unit 2 also.
4. Choice B is not plausible because more time is given for Unit 2 which is already in Mode 4.
5. Choice D is not plausible because the same amount of time is given for Unit 2 which is already in Mode 4. 6. Second bullet needs to give a stable RCS temperature instead of stating MODE 4 with a cooldown in progress.
7. Recommendation: use Unit 2 only and come up with four separate plausible times.

Q is U due to more than one non-plausible distracter.

04/18/2013

Facility Licensee modified Q as requested by NRC. Q appears SAT at this time.

5/09/2013

Facility requested to modify question following in-office review and validation. Additional comments below reflect the modified questions.

1. Q = K/A, Q = SRO only 2. Need to take out the time 0700. The time is irrelevant.
3. Need to discuss what the change did and how the previous question was not operationally valid.

05/21/2013 Facility licensee made changes as required. Q appears SAT at this time.

37 Q# 1. LOK (F/H) 2. LOD (1-5) 3. Psychometric Flaws

4. Job Content Flaws
5. Other 6. B/M/N 7. U/E/S 8. Explanation Stem Focus Cues T/F Cred. Dist. Partial Job-Link Minutia#/ units Back-ward Q= K/A SROOnly81 H 2 X Y N M E K/A 062AG2.1.23 1. Q=K/A, Question is not SRO only 2. First part of this question can be answered solely by knowing notes and cautions in procedures. ROs are required to know notes and cautions in procedures. ROs are also required to know RED/ORANGE path entry conditions. 3. Distracters A, C NP due to the reactor remaining at power.

Q is U due to more than one non-plausible distracter and license level mismatch.

04/18/2013

Facility Licensee discussed Q with NRC. NRC agreed with explanation provided by facility licensee. Q appears SAT at this time.

5/09/2013

Facility requested to modify question following in-office review and validation. Additional comments below reflect the modified questions.

1. Need to remove reason for orange path in FR-S.1 in fourth bullet (IRNI positive startup rate). This provides teaching in the stem. 2. Need to remove the reference to ten minutes in the fourth bullet and add that the crew is at the step for dispatching the AUO in the previous bullet.

05/21/2013

Facility licensee made changes as required. Q appears SAT at this time.

38 Q# 1. LOK (F/H) 2. LOD (1-5) 3. Psychometric Flaws

4. Job Content Flaws
5. Other 6. B/M/N 7. U/E/S 8. Explanation Stem Focus Cues T/F Cred. Dist. Partial Job-Link Minutia#/ units Back-ward Q= K/A SROOnly82 H 2 X X Y Y N E K/A 003AG2.2.36 1. Q=K/A, Q= SRO only 2. The second part question seems to provide a cue for the first part since there is no condition in the initial conditions that would be plausible to prevent recovery within one hour. 3. Need to specify which TS in the stem question.
4. Need to provide references for all > 1 hour1.157407e-5 days <br />2.777778e-4 hours <br />1.653439e-6 weeks <br />3.805e-7 months <br /> TS questions, and ensure the question is not a direct lookup.

04/18/2013

Facility Licensee modified Q as requested by NRC. Q still asks a greater than 1 hour1.157407e-5 days <br />2.777778e-4 hours <br />1.653439e-6 weeks <br />3.805e-7 months <br /> TS action from memory. NRC tabled the Q and will re-attack with another approach.

5/09/2013

Facility requested to modify question following in-office review and validation. Additional comments below reflect the modified questions.

1. Distracters A(1) and B(1) are not plausible when given the TS and reading the action statement which specifies the amount of power reduction required from Rated Thermal Power. This is essentially a direct lookup. The difference between current power and RTP and the choices given is 9% in each case. The previously supplied recommendation had a calculation that originated from 1.00 vice 1.02. 2. With the recovery in progress there is the possibility of no correct answer. 3. Need to provide conditions that specify the time till recovery will be accomplished. If it is greater than 24 hrs, will be required to be reduced to less than 50%.

Question U due to two non plausible distracters

05/21/2013 Facility licensee made changes as required. Q appears SAT at this time.

39 Q# 1. LOK (F/H) 2. LOD (1-5) 3. Psychometric Flaws

4. Job Content Flaws
5. Other 6. B/M/N 7. U/E/S 8. Explanation Stem Focus Cues T/F Cred. Dist. Partial Job-Link Minutia#/ units Back-ward Q= K/A SROOnly83 H 3 X Y Y N U K/A 028AG2.4.50 1. Q=K/A, Q= SRO only 2. Distracter B and C TS 3.0.3 actions (shutdown) are not plausible with only one instrument out of service. 3. Recommend basing the stem question on the current conditions, otherwise the first part of distracters C and D could be correct at a certain time.
4. Without giving the specifics of the first failure, it may not be plausible that the alarm came in for the first failure. 5. Need to provide references for all >1 hour TS questions, and ensure the question is not a direct lookup.

Q is U due to more than one non-plausible distracter.

5/09/2013 Facility requested to modify question following in-office review and validation. Additional comments below reflect the modified questions.

1. Question no longer meets the K/A.
2. Not operationally valid. Operators not required to remember all instrument numbers specified in TS tables. LOD = 5 3. Too many references
4. K/A rejected and replaced with K/A 028 AG2.4.4.47
5. Recommended replacement attached. Question is U due to no longer matching K/A and LOD = 5

05/21/2013

Facility licensee made changes as required. Q appears SAT at this time.

84 H 2 X X Y Y M E K/A 068AA2.06 1. Q=K/A, but Q is a weak K/A match on the SRO level. 2. Partial: distracter 'A' is a potentially correct answer because the timing of "prior to the cooldown" is not specific enough. Perhaps give a specific time associated with the first part question? 3. Benefit of doubt given on SRO link to K/A match; first part of the Q can be answered with RO level knowledge.

4. Spelling of "arrise" in the 0500 condition bullet.

04/18/2013

Facility Licensee modified Q as requested by NRC. Q appears SAT at this time.

40 Q# 1. LOK (F/H) 2. LOD (1-5) 3. Psychometric Flaws

4. Job Content Flaws
5. Other 6. B/M/N 7. U/E/S 8. Explanation Stem Focus Cues T/F Cred. Dist. Partial Job-Link Minutia#/ units Back-ward Q= K/A SROOnly85 H 4 X X X Y Y B U K/A W/E10EA2.2 1. Q=K/A, Q=SRO only.
2. Distracter B is not plausible because you cannot transition directly from ES-0.1 to ES-0.3. 3. Distracter D is not plausible because the restoration of offsite power is not guaranteed in the stem, and you do not know the leak rate in the CST.
4. As written, 'A' is a subset of 'C' and is potentially a correct answer. 5. Would it be operationally valid to require knowledge of the TS bases to the level of this Q without a reference?

Q is U due to more than one non-plausible distracter.

04/18/2013

Facility Licensee modified Q as requested by NRC. Q appears SAT at this time.

5/09/2013 Facility requested to modify question following in-office review and validation. Additional comments below reflect the modified questions.

1. Changing the answer to A would indicate that the question no longer matches the K/A 2. Bullet formatting needs to be corrected. 3. Need to explain why the answer changed.

Question is U due to no longer matching K/A

05/21/2013 Facility licensee made changes as required. Q appears SAT at this time.

41 Q# 1. LOK (F/H) 2. LOD (1-5) 3. Psychometric Flaws

4. Job Content Flaws
5. Other 6. B/M/N 7. U/E/S 8. Explanation Stem Focus Cues T/F Cred. Dist. Partial Job-Link Minutia#/ units Back-ward Q= K/A SROOnly86 H 2 X Y N N U K/A 003G2.4.11 1. Original K/A was rejected and replaced. 2. Q=K/A, not SRO-only knowledge. First part of Q can be answered with only RO level knowledge due to procedure section titles. 3. Second part of distracters A, B are potentially correct, and not independent of second part of distracters C, D; even if you are going to S/D and stop the pump in 8 hours9.259259e-5 days <br />0.00222 hours <br />1.322751e-5 weeks <br />3.044e-6 months <br />, wouldn't you also initiate a SR to repair/replace the seal?
4. There is an overlap issue with this question and question 5 on the RO exam; Q5 gives similar conditions and states that the conditions result in a #2 seal failure.

Q is U due to license level mismatch.

04/18/2013 Facility Licensee modified Q as requested by NRC. Q appears SAT at this time. 87 H 2 X Y Y B E K/A 010A2.02 1. Q=K/A, Q=SRO only. 2. Need to change the words "ineffectiv e" in both stem questions to "inadequate as defined by E-3" or reword and use the words "not adequate as defined by E-3" in order to match the words in the note and steps in E-3. This could also prevent interpretation problems with the word ineffective.

04/18/2013 Facility Licensee modified Q as requested by NRC. Q appears SAT at this time. 88 F 2 Y Y B S K/A 012A2.05 1. Q=K/A, Q=SRO only

Q appears SAT at this time.

05/21/2013

Minor typographical changes provided to licensee for initial conditions and stem questions. Q appears SAT at this time.

89 F 3 X Y Y N U K/A 025G2.2.25 1. Q=K/A, Q=SRO only 2. Multiple correct answers. Sodium tetraborate solution assists in maintaining a basic pH of 9.0 to 9.5, which minimizes the occurrence of corrosion. Need to develop another distracter for the second part Q.

Q is U due to multiple correct answers.

04/18/2013

Facility Licensee modified Q as requested by NRC. Q appears SAT at this time.

42 Q# 1. LOK (F/H) 2. LOD (1-5) 3. Psychometric Flaws

4. Job Content Flaws
5. Other 6. B/M/N 7. U/E/S 8. Explanation Stem Focus Cues T/F Cred. Dist. Partial Job-Link Minutia#/ units Back-ward Q= K/A SROOnly90 F 2 X X Y Y N E K/A 073A2.01 1. Original K/A was rejected and replaced. 2. Q=K/A, Q=SRO only based on ODCM actions greater than one hour.
3. First part of Q is RO-level knowledge of automatic rad monitor actions. 4. Need to provide references for all questions asking application of greater than one hour specifications, and ensure that the resulting Q statement does not result in a direct

look-up. 04/18/2013

Facility Licensee modified Q as requested by NRC. Q appears SAT at this time. 91 H 2 X X Y Y N E K/A 014A2.04 1. Q=K/A, Q=SRO only

2. Grammatically awkward statements in third and fifth bullets, need to make them more clearly stated. Misspelled "conditions" in Q statement. 3. As written, B and D are also potentially correct answers because taking those actions would result in meeting the requirements of the TS LCO. 4. A is a non-plausible distracter. There are no IRPIs in the question that have failed, and TS statements are not written with the first part statements giving a longer completion time than the alternative (or) action. Need to develop a plausible alternative to the 'A' distracter. 5. Potential fixes: specifically ask what the TS required action(s) is/are as stated in the TS, instead of what actions are required. Ensure the distracters are all logically independent of each other.

04/18/2013

Facility Licensee modified Q as requested by NRC. Q appears SAT at this time.

05/21/2013 Minor typographical changes provided to licensee for initial conditions. Q appears SAT at this time. 92 H 4 Y Y B S K/A 028A2.03 1. Q=K/A, Q=SRO only.

Q appears SAT at this time.

05/21/2013

Minor typographical changes provided to licensee for initial conditions and stem questions. Q appears SAT at this time.

43 Q# 1. LOK (F/H) 2. LOD (1-5) 3. Psychometric Flaws

4. Job Content Flaws
5. Other 6. B/M/N 7. U/E/S 8. Explanation Stem Focus Cues T/F Cred. Dist. Partial Job-Link Minutia#/ units Back-ward Q= K/A SROOnly93 F 2 X Y Y N U K/A 055G2.4.30 1. Original K/A was rejected and replaced. 2. Q=K/A, Q=SRO only.
3. First part of the Q is RO level knowledge.
4. Second part of this Q overlaps with Admin JPM SRO A.1.a with notifications-SM is only required to notify Site Ops management and Duty Plant Manager. Need to develop another approach to second part of this Q or the admin JPM.

Q is U due to excessive overlap with op. test.

04/18/2013 Facility Licensee completely changed out the administrative JPM to remove the overlap concern. Q appears SAT at this time.

05/21/2013

Minor typographical changes provided to licensee for the stem statement and stem questions. Q appears SAT at this time. 94 F 3 X Y Y B E K/A G2.1.4 1. In second part Q statement, need to change "-Shift Manager shall arrange for-" to "-Shift Manager is required to arrange for-"; recommend underline/bold "maximum" 2. This Q was asked on the Dec 2012 NRC exam.

04/18/2013

Facility Licensee modified Q as requested by NRC. Q appears SAT at this time.

05/21/2013

Minor typographical changes provided to licensee for stem statement. Q appears SAT at this time. 95 H 3 X Y Y B E K/A G2.2.17 1. Q=K/A, Q=SRO only 2. Recommend providing entire procedure NPG-SPP-07.3 as a reference for this Q.

04/18/2013 Facility Licensee modified Q as requested by NRC. Q appears SAT at this time.

44 Q# 1. LOK (F/H) 2. LOD (1-5) 3. Psychometric Flaws

4. Job Content Flaws
5. Other 6. B/M/N 7. U/E/S 8. Explanation Stem Focus Cues T/F Cred. Dist. Partial Job-Link Minutia#/ units Back-ward Q= K/A SROOnly96 H 3 X X Y Y M E K/A G2.2.36 1. Original K/A was rejected and replaced. 2. Distracter analysis provided for this question does not support A as the correct answer. 3. Need to provide references for all questions asking application of greater than one hour specifications, and ensure that the resulting Q statement does not result in a direct

look-up. 4. Potential fix: change second part of Q to ask when the latest time the first performance of O-SI-OPS-082-007.W is required to be completed, and use 1 hr vs. 1.25 hrs as the two distracters (i.e. applying the 25% SR extension frequency).

04/18/2013

Facility Licensee modified Q as requested by NRC. Q appears SAT at this time. 97 H 3 X X Y Y B E K/A G2.3.6 1. Q=K/A, Q=SRO only.

2. Spelling error "addtional" in distracter 'B' 3. Partial/cred distracters: C, D are not logically independent of each other and grammatically the "NOT permitted" phrasing does not work with the Q statement. Recommend revising as follows: "Provided ODCM compliance is maintained, this Monitor Tank release is permitted __________________ .

A. only after the Shift Manager obtains the Operations Superintendent approval

B. after the US/SRO grants approval, without any additional signatures/approvals needed.

C. only after the contents of the Monitor Tank are reprocessed to lower activity

D. only after 0-RM-90-122 has been returned to an OPERABLE status" 04/18/2013

Facility Licensee modified Q as requested by NRC. Q appears SAT at this time. 98 F 3 Y Y M S K/A G2.3.7 1. Q=K/A, Q=SRO only

Q appears SAT at this time.

45 Q# 1. LOK (F/H) 2. LOD (1-5) 3. Psychometric Flaws

4. Job Content Flaws
5. Other 6. B/M/N 7. U/E/S 8. Explanation Stem Focus Cues T/F Cred. Dist. Partial Job-Link Minutia#/ units Back-ward Q= K/A SROOnly99 H 2 X X X Y Y B E K/A G2.4.18 1. Original K/A was rejected and replaced. 2. Q=K/A, Q=SRO only.
3. Overlap: As currently written, Q 99 provides a way to eliminate two distracters of RO Q 49 (the fact that no operator actions would be needed to strip DC busses on ECA-0.0 situation).
4. Distracter A is not plausible as currently written. The second part Q statement provided a cue that 250V DC loads would be shed using EA-250-2. 5. Need to remove "-when shedding 250V DC loads" from second part Q statement (teaching in stem)
6. Need to remove "-turbine is at zero speed-" from second part distracters B and D to ensure they are the correct choice (procedure mainly discusses turbine heat load)

04/18/2013 Facility Licensee modified Q as requested by NRC. Q appears SAT at this time.

05/21/2013

Minor typographical changes provided to licensee for stem statement. Q appears SAT at this time. 100 F 2 X Y Y B E K/A G2.4.29 1. Q=K/A, Q=SRO only

2. Need to change "-the SED is to-" to "-the SED is required to-" 3. Recommend change: " - and notification is/is NOT required to be made to the State of Tennessee." for second part of Q.
4. On p. 21 of NPG-SPP-03.5, which is provided as a reference to Q98, the NOTE following 3.1 B. step 2 provides the answer to the first part of Q100. When this procedure is given as a reference, information from section 3.1 B must be fully redacted [contact examiners if assistance is needed on .pdf redaction].

04/18/2013 Facility Licensee modified Q as requested by NRC. Q appears SAT at this time.

46